Tw. Viggo Bruna, "Wstęp..." W. Sierpińskiego

Podzielność. Reszty z dzielenia. Kongruencje. Systemy pozycyjne. Równania diofantyczne. Liczby pierwsze i względnie pierwsze. NWW i NWD.
patry93
Użytkownik
Użytkownik
Posty: 1251
Rejestracja: 30 sty 2007, o 20:22
Płeć: Mężczyzna
Lokalizacja: Koziegłówki/Wrocław
Podziękował: 352 razy
Pomógł: 33 razy

Tw. Viggo Bruna, "Wstęp..." W. Sierpińskiego

Post autor: patry93 »

Witam.

Mam problem z dowodem (dokładniej, ze zrozumieniem części gotowego dowodu) twierdzenia Viggo Bruna (znaleźć je można np. tu - ), który znajduje się w książce "Wstęp do teorii liczb" W. Sierpińskiego (str. 61-62, gdyby ktoś posiadał ). Nie będę raczej przepisywał całego dowodu, tylko poszczególne jego części, które sprawiają mi problem, natomiast resztę tylko pokrótce opiszę (mam nadzieję, że będzie to wystarczające do zrozumienia "o co właściwie chodzi" ).

Na początku autor twierdzi, że dane tw. można uogólnić na dowolny rosnący i nieskończony ciąg liczb naturalnych \(\displaystyle{ p_1, p_2, \ldots \ (1)}\) (niestety nie bardzo widzę, czy jest to podawane bez dowodu, czy może dalsza część już nim jest (dowodem ), ponieważ mam problem ze zrozumieniem "co do czego"), dalej dowodzony jest wzór \(\displaystyle{ \pi (a+b) \le \pi(a)+b}\) przy założeniu, że \(\displaystyle{ \pi(x)}\) oznacza liczbę tych wyrazów ciągu \(\displaystyle{ (1)}\), które są \(\displaystyle{ \le x}\).
Dalej korzystając z udowodnionego wzoru, pokazane jest, że
\(\displaystyle{ n_{i+1} = n - \pi(n_0 + n_1 + \ldots + n_i) \ge 0 \ (2)}\), gdzie \(\displaystyle{ n_0=n}\).
Dalej, zakładając, że wszystkie wyrazy ciągu są dodatnie, przy dowolnym naturalnym \(\displaystyle{ s}\) byłoby
\(\displaystyle{ n_1+n_2+ \ldots + n_{s-1} \ge s}\)
natomiast moim zdaniem, powinno być \(\displaystyle{ n_0+n_1+ \ldots + n_{s-1} \ge s}\) - czy mam rację?
(dalej będę pisał z moją modyfikacją)
skąd \(\displaystyle{ \pi (n_0+n_1+ \ldots + n_{s-1}) \ge \pi(s)}\)
i wobec \(\displaystyle{ (2)}\) dla \(\displaystyle{ i=s-1}\), jest \(\displaystyle{ n \ge \pi (n_0+n_1+ \ldots + n_{s-1}) \ge \pi(s)}\)
skąd w szczególności dla \(\displaystyle{ s=p_{n+1}}\), wobec \(\displaystyle{ \pi(p_{n+1}) =n+1}\), byłoby \(\displaystyle{ n+1 \le n}\) - sprzeczność, więc istnieje wyraz zerowy, tj.
\(\displaystyle{ n - \pi(n_0 + n_1 + \ldots + n_k) =0}\)
i zakładamy, że \(\displaystyle{ k}\) jest najmniejszą liczbą o tej własności.
Gdyby \(\displaystyle{ k=0}\), to wobec \(\displaystyle{ n_0=n}\), jest \(\displaystyle{ n= \pi(n)}\) (tutaj kompletnie nie rozumiem, po co dalsze rozważania, skoro... nigdy nie zachodzi \(\displaystyle{ x= \pi(x)}\)?), co pociąga za sobą \(\displaystyle{ p_i=i}\) dla \(\displaystyle{ i \le n}\) (tego za nic nie potrafię rozgryźć, dlaczego tak :/) skąd \(\displaystyle{ p_n=n}\) i teza byłaby prawdziwa (kolejna dla mnie zagadka).
Dalsze rozważania tyczą się \(\displaystyle{ k>0}\), ale tego nie będę pisał, ponieważ lepiej byłoby najpierw początek zrozumieć.

Z góry dziękuję.
ODPOWIEDZ